Đến nội dung

Hình ảnh

$\sum_{sym}^{a,b,c,d}\frac{1}{a^3+b^3}\geq \frac{243}{2(a+b+c+d)^3}$


  • Please log in to reply
Chủ đề này có 1 trả lời

#1
duy1237

duy1237

    Lính mới

  • Thành viên
  • 3 Bài viết
Cho a,b,c,d thực không âm, trong đó không có hai số nào đồng thời bằng 0.
Chứng minh:
$\frac{1}{a^3+b^3}+\frac{1}{a^3+c^3}+\frac{1}{a^3+d^3}+\frac{1}{b^3+c^3}+\frac{1}{b^3+d^3}+\frac{1}{c^3+d^3}\geq \frac{243}{2(a+b+c+d)^3}$

#2
minhlaai29

minhlaai29

    Binh nhất

  • Thành viên
  • 20 Bài viết
Vì bất đẳng thức đã cho là thuần nhất nên ta có thể chuẩn hóa $a+b+c+d=1$.
Không mất tính tổng quát giả sử $d=$ min {$a,b,c,d$}(*)
Đặt $f(a,b,c,d)=\sum_{sym}^{a,b,c,d}\frac{1}{a^{3}+b^{3}}$
Ta sẽ chứng minh rằng $\sum_{sym}^{a,b,c,d}\frac{1}{a^{3}+b^{3}}\geq\sum_{cyc}^{a,b,c}\frac{1}{(a+\frac{d}{3})^{3}+(b+\frac{d}{3})^{3}} + \sum_{cyc}^{a,b,c}\frac{1}{(a+\frac{d}{3})^{3}}$
Ta có các bất đẳng thức sau:
$a^{3}+b^{3}\leq (a+\frac{d}{3})^{3}+(b+\frac{d}{3})^{3}$ (1)
$a^{3}+c^{3}\leq (a+\frac{d}{3})^{3}+(c+\frac{d}{3})^{3}$ (2)
$b^{3}+c^{3}\leq (b+\frac{d}{3})^{3}+(c+\frac{d}{3})^{3}$ (3)
$a^{3}+d^{3}\leq (a+\frac{d}{3})^{3}$ (4)
$b^{3}+d^{3}\leq (b+\frac{d}{3})^{3}$ (5)
$c^{3}+d^{3}\leq (c+\frac{d}{3})^{3}$ (6)
Các bất đẳng thức (1) (2) (3) hiển nhiên đúng vì $a,b,c,d\geq 0$
Các bất đẳng thức (4) (5) (6) chứng minh tương tự nhau nên ta chỉ cần chứng minh bất đẳng thức (4).
Khai triển vế phải ta đưa bất đẳng thức (4) về dạng tương đương:
$a^{2}d + \frac{ad^{2}}{3}\geq \frac{26}{27}d^{3}$
Bất đẳng thức trên đúng do (*). Vậy ta chứng minh được các bất đẳng thức (4) (5) (6)
Áp dụng các bất đẳng thức (1)(2)(3)(4)(5)(6) ta có :
$\sum_{sym}^{a,b,c,d}\frac{1}{a^{3}+b^{3}}\geq\sum_{cyc}^{a,b,c}\frac{1}{(a+\frac{d}{3})^{3}+(b+\frac{d}{3})^{3}} + \sum_{cyc}^{a,b,c}\frac{1}{(a+\frac{d}{3})^{3}}$
$\Leftrightarrow f(a,b,c,d)\geq f(a+\frac{d}{3};b+\frac{d}{3};c+\frac{d}{3};0)$
Vậy ta chỉ còn phải chứng minh bất đẳng thức trong trường hợp $d=0$ và $a+b+c=1$ thì:
$\sum_{cyc}^{a,b,c}\frac{1}{a^{3}+b^{3}}+\sum_{cyc}^{a,b,c}\frac{1}{a^{3}}\geq \frac{243}{2}$
$\Leftrightarrow 2(a^{3}+b^{3}+c^{3})\left (\frac{1}{(a^{3}+b^{3})c^{3}}+\frac{1}{(c^{3}+b^{3})a^{3}}+\frac{1}{(a^{3}+c^{3})b^{3}} \right )\geq 243$
$\Leftrightarrow 2(a+b+c)^{3}(a^{3}+b^{3}+c^{3})\left (\frac{1}{(a^{3}+b^{3})c^{3}}+\frac{1}{(c^{3}+b^{3})a^{3}}+\frac{1}{(a^{3}+c^{3})b^{3}} \right )\geq 243$
Áp dụng bất đẳng thức Holder cho các bộ số $(a,b,c) ; (c^{3}+b^{3},a^{3}+c^{3},a^{3}+b^{3});\left (\frac{1}{(c^{3}+b^{3})a^{3}},\frac{1}{(c^{3}+a^{3})b^{3}},\frac{1}{(a^{3}+b^{3})c^{3}} \right )$ ta có điều phải chứng minh.
Dấu "=" xảy ra khi và chỉ khi $a=b=c,d=0$ hoặc các hoán vị tương ứng.

Bài viết đã được chỉnh sửa nội dung bởi minhlaai29: 13-12-2012 - 00:12





1 người đang xem chủ đề

0 thành viên, 1 khách, 0 thành viên ẩn danh